2
$\begingroup$

Suppose $P$ is a finitely additive probability measure on a space $\Omega$, and $X_1,X_2,X_3,\dots$ are i.i.d. random variables with mean 0 and variance 1. Is it true that for all $r \in \mathbb{R}$, $$\lim_{n\to\infty} P \left( \sqrt{n}\sum_{i = 1}^n X_i \leq r \right) = \frac{e^{-r^2/2}}{\sqrt{2\pi}}?$$

Basically I am asking whether countable additivity plays an essential role in the proof of the classical CLT.

See this question for a brief discussion of random variables in the finitely additive setting.

$\endgroup$
3
  • $\begingroup$ Is this question not solved in doi.org/10.1016/0047-259X(88)90035-8 , Theorem 2.1? In particular use ch.3, Consequences (i) of this paper. $\endgroup$ Commented Aug 27 at 9:24
  • 1
    $\begingroup$ @DieterKadelka Thanks for pointing that out. Not sure but I think the statement of the CLT implicitly assumes that the integral of $X_1$ (the mean) exists according to the definition in that paper, which means that there is a kind of "Cauchy sequence" of simple functions converging to $X_1$ in measure. In the finitely additive setting, this is not, apriori, necessarily the broadest class of measurable functions for which a well-defined integral exists. So if my understanding is correct, he solves the question affirmatively for his $L_1$ random variables. Good enough for most purposes. $\endgroup$ Commented Aug 27 at 11:32
  • $\begingroup$ A good place to check would be "How to gamble if you must" by Dubins and Savage. I don't have a copy on hand, so I cannot look at it myself. $\endgroup$ Commented Aug 27 at 15:07

0

You must log in to answer this question.

Start asking to get answers

Find the answer to your question by asking.

Ask question

Explore related questions

See similar questions with these tags.